Difference between revisions of "2008 AMC 12A Problems/Problem 2"
m (wikified) |
I like pie (talk | contribs) (Standardized answer choices) |
||
Line 2: | Line 2: | ||
What is the [[reciprocal]] of <math>\frac{1}{2}+\frac{2}{3}</math>? | What is the [[reciprocal]] of <math>\frac{1}{2}+\frac{2}{3}</math>? | ||
− | <math>\ | + | <math>\mathrm{(A)}\ \frac{6}{7}\qquad\mathrm{(B)}\ \frac{7}{6}\qquad\mathrm{(C)}\ \frac{5}{3}\qquad\mathrm{(D)}\ 3\qquad\mathrm{(E)}\ \frac{7}{2}</math> |
==Solution== | ==Solution== |